Partitions involving D. H. Lehmer numbers (Q2655196): Difference between revisions

From MaRDI portal
RedirectionBot (talk | contribs)
Changed an Item
ReferenceBot (talk | contribs)
Changed an Item
 
(2 intermediate revisions by 2 users not shown)
Property / MaRDI profile type
 
Property / MaRDI profile type: MaRDI publication profile / rank
 
Normal rank
Property / full work available at URL
 
Property / full work available at URL: https://doi.org/10.1007/s00605-008-0049-z / rank
 
Normal rank
Property / OpenAlex ID
 
Property / OpenAlex ID: W1965894940 / rank
 
Normal rank
Property / cites work
 
Property / cites work: On Kloosterman's sum / rank
 
Normal rank
Property / cites work
 
Property / cites work: Q3037553 / rank
 
Normal rank
Property / cites work
 
Property / cites work: On the generalization of the D. H. Lehmer problem / rank
 
Normal rank
Property / cites work
 
Property / cites work: Q2785542 / rank
 
Normal rank
Property / cites work
 
Property / cites work: Q4694932 / rank
 
Normal rank
Property / cites work
 
Property / cites work: Q4296334 / rank
 
Normal rank

Latest revision as of 10:23, 2 July 2024

scientific article
Language Label Description Also known as
English
Partitions involving D. H. Lehmer numbers
scientific article

    Statements

    Partitions involving D. H. Lehmer numbers (English)
    0 references
    0 references
    0 references
    22 January 2010
    0 references
    Let \(n \geq 2\) be a fixed integer, let \(q\) and \(c\) be two integers with \(q > n\) and \((n, q) = (c, q) = 1\). For every positive integer \(a\) which is coprime with \(q\) we denote by \(\overline{a}_c\) the unique integer satisfying \(1\leq\overline{a}_c \leq q\) and \(a\overline{a}_c \equiv c\pmod q\). Put \[ L(q)=\{a\in\mathbb Z^{+}: (a,q)=1,\, n \mid a+\overline{a}_c \}. \] The elements of \(L(q)\) are called D. H. Lehmer numbers. In this paper the authors prove that every sufficiently large integer can be expressed as the sum of three D. H. Lehmer numbers and the number of the representations of \(N\) is \[ \frac{N^2}{2}\left(1-\frac 1n\right)^3 \frac{\varphi^3(q)}{q^3}\prod_{p\mid (q,N)}\left(1-\frac 1{(p-1)^2}\right) \prod_{p\mid q,\, p\nmid N} \left(1+\frac 1{(p-1)^3}\right)+O(N^2q^{-1/2}d^9(q)\log^3q), \] where the constant \(O\) depends only on \(n\) and \(d(q)\) is the divisor function.
    0 references
    0 references
    0 references
    0 references
    0 references
    0 references
    0 references
    0 references